4
$\begingroup$

Fix a prime $p$ and let $G$ be a connected reductive group over a $p$-adic local field $K$. Let $H \subset G$ be a connected reductive subgroup such that $G(K)/H(K)$ is compact in the $p$-adic topology. Under what assumptions on $G$ must $H$ be equal to $G$?

Some assumptions are clearly required, since $G(K)$ itself could be compact. So certainly we need to assume that all factors of $G^{\mathrm{ad}}(K)$ are totally isotropic. We also need to assume that the center $Z_G$ of $G$ is split, because otherwise $G$ could be a torus and $H$ the maximal $K$-split subtorus.

A theorem of Borel--Tits (Proposition 9.3 of "Groupes réductifs") tells us that H must contain a "sous-groupe connexe trigonalisable sur k maximal". When $G$ is split, this is just a Borel subgroup of $G$ and therefore $H$ is a reductive subgroup of $G$ containing a Borel, implying that $H=G$ (this is a well known argument).

I suspect that it is not necessary that $G$ is split. If $G = \operatorname{Res}_{F/K} Q_{F}$ with $Q$ split over $k$, then a "sous-groupe connexe trigonalisable sur k maximal" is probably of the form $\operatorname{Res}_{F/K} U \rtimes T$, where $B=TU$ is a Borel of $Q$ over $K$ with $T$ split. I suspect that a reductive subgroup $H$ of $G$ containing $\operatorname{Res}_{F/K} U \rtimes T$ must equal all of G, but I don't see how to prove that.

$\endgroup$
2
  • 3
    $\begingroup$ I think it happens only when $H$ contains a normal $K$-subgroup such that $G/H$ is anisotropic (even assuming only $H$ unimodular instead of reductive). So for given $G$ there non such proper subgroup iff its connected center is split and all its simple factors are isotropic. $\endgroup$ Commented Mar 17 at 7:47
  • 2
    $\begingroup$ (Borel-Tits applies when $H$ is not assumed reductive/unimodular and thus addresses much less trivial cases.) $\endgroup$ Commented Mar 17 at 7:50

1 Answer 1

4
$\begingroup$

From the comments, it sounds like there might exist a reference for a positive answer to this question. However, I don't know it, so here is an argument using Borel-Tits.

Claim. The maximal smooth connected trigonalizable $K$-subgroups of $G$ are of the form $S \cdot U_G(\lambda)$, where $S$ is a maximal split $K$-torus of $G$ and $\lambda\colon \mathbf{G}_m \to S$ is a regular cocharacter, i.e., $\langle\lambda, \alpha\rangle \neq 0$ for all roots $\alpha$ of $(G, S)$ (and $U_G(\lambda)$ is the notation from the dynamic method).

Proof. First, it is well-known that $S \cdot U_G(\lambda)$ is trigonalizable. By Theorem 8.2 of Borel-Tits, all maximal trigonalizable $K$-subgroups of $G$ are $G(K)$-conjugate, so it's enough to show that $S \cdot U_G(\lambda)$ is maximal. The multiplication map $U_G(\lambda^{-1}) \times Z_G(S) \times U_G(\lambda) \to G$ is an open embedding, so for dimension reasons we need only show that if $V$ is a trigonalizable $K$-subgroup of $G$ containing $S \cdot U_G(\lambda)$ then $(V \cap Z_G(S))/S$ and $V \cap U_G(\lambda^{-1})$ are both finite.

Since $Z_G(S)/S$ is $K$-anisotropic, $S$ is the maximal smooth connected trigonalizable $K$-subgroup of $Z_G(S)$ and thus $(V \cap Z_G(S))/S$ is finite. Moreover, if $V \cap U_G(\lambda^{-1})$ is nontrivial then the Lie algebra of $V$ contains a pair of vectors with opposite $S$-weights, and this cannot happen for a trigonalizable $K$-subgroup of $G$ (reduce to $K = \overline{K}$ and think about a Borel). QED

The question is not sensitive to isogenies, so we may assume $G = Z \times G'$, where $Z$ is a split torus and $G'$ is semisimple and simply connected. By the Claim and Proposition 9.3 of Borel-Tits, $H$ must contain $S \cdot U_G(\lambda)$ for some $S$ and $\lambda$. In particular, $H$ contains $Z$, so we reduce to the case $G = G'$. Now $H$ must also contain $S \cdot U_G(\lambda^{-1})$: indeed, because $H$ is reductive, $\dim U_H(\lambda) = \dim U_H(\lambda^{-1})$. Since also $\dim U_G(\lambda) = \dim U_G(\lambda^{-1})$ and $U_G(\lambda) = U_H(\lambda)$, we get the claim by a dimension argument. (This is the only place where reductivity of $H$ is used. It probably only requires that $H(K)$ be unimodular, as YCor suggested, but I haven't checked carefully.)

Thus the problem reduces to showing that $G$ is generated by $U_G(\lambda)$ and $U_G(\lambda^{-1})$. Note that $G = \prod_{i=1}^n \mathrm{R}_{K_i/K}(G_i)$ for finite separable field extensions $K_i/K$ and absolutely simple $K_i$-groups $G_i$, and $U_G(\lambda^{\pm 1})$ decomposes into a product of subgroups of the $\mathrm{R}_{K_i/K}(G_i)$, so we may assume $n = 1$ (and of course each $G_i$ is isotropic since you are assuming $G$ is totally isotropic). We have that $S$ is the maximal $K$-split torus of $\mathrm{R}_{K_1/K}(S_1)$ for a unique maximal $K_1$-split torus $S_1$ of $G_1$. If $\lambda_1\colon \mathbf{G}_m \to S_1$ is the cocharacter corresponding to $\lambda$, then we have $U_G(\lambda^{\pm 1}) = \mathrm{R}_{K_1/K}(U_{G_1}(\lambda_1^{\pm 1}))$ (just think about weights on the Lie algebra), so it is enough to show that $G_1$ is generated by $U_{G_1}(\lambda_1^{\pm 1})$; in other words, we may assume that $G_1$ is absolutely simple.

The question of whether $G$ is generated by $U_G(\lambda^{\pm 1})$ is purely algebraic, so by passing to $\overline{K}$ it would in fact be enough to show that if $G$ is split simple and $\lambda$ is any nontrivial cocharacter then $G$ is generated by $U_G(\lambda^{\pm 1})$. If $\lambda$ factors through a (split) maximal $K$-torus $T$, then it is enough to show that $G$ is generated by $T$ and $U_G(\lambda^{\pm 1})$: indeed, if $G_0$ is the $K$-subgroup generated by $U_G(\lambda^{\pm 1})$, then $G_0$ is normalized by $T$, so if $G = \langle G_0, T \rangle$ then $G_0$ is normal in $G$, and by simplicity $G = G_0$. Thus we have reduced to the following root-theoretic claim: if $(V, \Phi)$ is an irreducible root system and $f\colon V \to \mathbf{R}$ is a nontrivial linear functional, then $\mathbf{Z}\Phi$ is spanned by the set of roots $a$ such that $f(a) \neq 0$.

For this, note that there is a basis $B$ of $(V, \Phi)$ such that $f(b) \geq 0$ for all $b \in B$. Since $f$ is nontrivial, there is some $a \in B$ such that $f(a) > 0$. It's a general fact about root systems (see Bourbaki, Chap. VI, no. 1.6, Corollaire 3) that if $C$ is a connected subgraph of the Dynkin diagram, then $s_C = \sum_{b \in C} b$ is a root. It is easy to see that $\mathbf{Z}\Phi$ is spanned by $s_C$ as $C$ ranges over connected subgraphs of the Dynkin diagram containing $a$ (to get another $b \in B$, induct on the length of a shortest path from $a$ to $b$), and of course $f(s_C) > 0$ for each such $C$. So we're done!

$\endgroup$
4
  • 2
    $\begingroup$ Should "$V \cap U_G(\lambda^{-1})$ is nontrivial" pass to the identity component before testing for triviality? $\endgroup$ Commented Mar 17 at 14:44
  • 2
    $\begingroup$ I think it doesn't matter: $K$ is of characteristic 0 and $U_G(\lambda^{-1})$ is unipotent, so $V \cap U_G(\lambda^{-1})$ is automatically smooth and connected. (That said, I don't think the assumption that $K$ is of characteristic 0 was used very seriously, and this should also apply to function fields.) $\endgroup$ Commented Mar 17 at 14:54
  • $\begingroup$ This is great, thank you! $\endgroup$ Commented Mar 17 at 16:51
  • 1
    $\begingroup$ Re, ah, right! If I had a nickel for every time I've worried about disconnectedness of unipotent groups because I forgot we were in characteristic $0$, I'd have two nickels, which isn't a lot, but etc. $\endgroup$ Commented Mar 17 at 17:52

You must log in to answer this question.

Start asking to get answers

Find the answer to your question by asking.

Ask question

Explore related questions

See similar questions with these tags.